LSAT and Law School Admissions Forum

Get expert LSAT preparation and law school admissions advice from PowerScore Test Preparation.

 Administrator
PowerScore Staff
  • PowerScore Staff
  • Posts: 8916
  • Joined: Feb 02, 2011
|
#34704
Complete Question Explanation

Assumption—CE. The correct answer choice is (B)

This stimulus contains a fairly complicated argument containing an intermediate conclusion. The
author tells us that young people’s pessimism concerning the potential success of efforts to reduce
pollution, poverty, and war is probably harmful to humanity’s future, because this pessimism will
reduce their motivation to work toward the goals of reducing pollution, poverty and war. Based on
this view, the author tells us that we have to do what we can to keep young people from losing their
motivation, and concludes that we have to “enable our children to believe that better futures are
possible.”

The question stem identifies this as an Assumption question. The correct answer choice will contain
information logically required for the conclusion to be valid. In this case, the conclusion was that we
have to “enable our children to believe that better futures are possible.” The primary reason offered
for this conclusion was the need to keep young people from losing their motivation. However, we
do not have any reason to think that enabling children to believe that better futures are possible will
have this effect. The author is assuming that this is the case.

So, our prephrase is that the author assumes that “enabling children to believe that better futures
are possible” will prevent children from losing their motivation. Looking back to the rest of the
argument, our prephrase is strengthened by the author’s earlier statement that pessimism reduces
motivation. Apparently, the author thinks that “enabling children to believe that better futures are
possible” will keep them from becoming pessimistic, and therefore prevent the negative effect that
pessimism can have on motivation.

Answer choice (A): This answer choice is incorrect because it jumbles up the various pieces of the
stimulus. Instead of motivation reducing pessimism, the argument assumed that belief in the future
will reduce pessimism which will prevent a reduction in motivation.

Answer choice (B): This is the correct answer choice because it correctly identifies the author’s
assumption about the connection between people’s beliefs in the future, pessimistic beliefs, and
motivation.

Answer choice (C): The concept of an “illusory vision” about the future had no role in the author’s
argument. Although the author urges us to help people believe in a better future, that does not mean
that the “better future” is illusory.

Answer choice (D): The conclusion does not involve what will actually occur in the future, so this
information is not required for the conclusion to be valid.

Answer choice (E): Similarly to answer choice (D), the argument made no mention of what occurred
in the past, so this information is irrelevant to the conclusion.

Get the most out of your LSAT Prep Plus subscription.

Analyze and track your performance with our Testing and Analytics Package.